LSAT and Law School Admissions Forum

Get expert LSAT preparation and law school admissions advice from PowerScore Test Preparation.

 eober
  • Posts: 107
  • Joined: Jul 24, 2014
|
#16403
Hi,

Is the reason why answer choice C is not weakening the argument is because it is not attacking the conclusion, which relates to the legislation?

Thanks!
 David Boyle
PowerScore Staff
  • PowerScore Staff
  • Posts: 836
  • Joined: Jun 07, 2013
|
#16427
eober wrote:Hi,

Is the reason why answer choice C is not weakening the argument is because it is not attacking the conclusion, which relates to the legislation?

Thanks!
Hello,

Answer C does mention the legislation, but, perhaps among other things, mentions injuries in general, not serious injuries as the stimulus mentions.

David
 Sherry001
  • Posts: 81
  • Joined: Aug 18, 2014
|
#20265
Hello ;
I totally see why A is the answer for this lovely question. But just had a quick question with C. Could you please see my reasoning below and let know what you think ?


1- legislation gave gov increased control over industrial work place.
2- among high risk jobs the likelihood that a worker would injur has decreased.
C: legislation has increased the overall safety within high risked industries.


C) the annual number of work related injuries has increased since legislation took effect.

Okay, so I saw this as wrong for two reasons : first, that it's concerned with all kinds of safety , but we only care about the high risked industries. Second , I also felt this strengthened the argument a bit because the stimulus tells us that there has been a drop in percentage while this tells us that the numbers have gone up .
Am I crazy for seeing this ??

Thanks so much ,
Sherry
 BethRibet
PowerScore Staff
  • PowerScore Staff
  • Posts: 200
  • Joined: Oct 17, 2012
|
#20268
Hi Sherry,

Thanks for the question. You're on the right track. I think the biggest flaw in answer choice C touches on what appears to be your second point. That is, the answer choice is about quantity/numbers, and the stimulus is about likelihood/percentage. In other words, this answer choice appears to equate number with percentage, which as you likely know, is a classic flaw on the LSAT.

Hope this helps!

Beth
 lsat2016
  • Posts: 59
  • Joined: May 29, 2016
|
#25743
Hello,

Could you explain why answer choice E is incorrect?? Since the answer choice is broader in scope than specifically high-risk industries, shouldn't it weaken the argument??

Thank you!!
 Emily Haney-Caron
PowerScore Staff
  • PowerScore Staff
  • Posts: 577
  • Joined: Jan 12, 2012
|
#25865
Hi lsat2016,

E doesn't weaken the argument because the legislation gave the government increased control over safety conditions even in non-high wish industries. Even with E, then, the legislation could be the cause of the generally improved conditions across industries.

Does that help?
 lsat2016
  • Posts: 59
  • Joined: May 29, 2016
|
#25867
Emily Haney-Caron wrote:Hi lsat2016,

E doesn't weaken the argument because the legislation gave the government increased control over safety conditions even in non-high wish industries. Even with E, then, the legislation could be the cause of the generally improved conditions across industries.

Does that help?
Thanks for your reply! Are you saying that E raises the possibility that the legislation might give the government increased control only over non-high risk industries and not high-risk industries?
 Nikki Siclunov
PowerScore Staff
  • PowerScore Staff
  • Posts: 1362
  • Joined: Aug 02, 2011
|
#25869
Hi lsat2016,

Welcome to the Forum!! :-D

There is no reason to suspect that (A) weakens the argument. The argument is essentially causal: the author observes a decrease in likelihood of serious injury since 1955, which coincides with the introduction of the new legislation. From this, the author concludes that the new legislation was responsible for the increased worker safety:
  • ..... ..... ..... Cause ..... ..... Effect

    ..... New Legislation :arrow: Increased safety
This may be so, but answer choice (A) presents an alternative cause: technological innovation! This is an alternative cause that could easily explain the increased worker safety, weakening the argument.

Answer choice (E), by contrast, has no relation to this causal argument. Maybe workplace safety conditions have improved across all industries - so what? This could be the result of the same legislation that the author believes improved worker safety in the high-risk industries. If anything, since the legislation does not appear to target high-risk industries in particular, answer choice (E) can easily be interpreted as strengthening the conclusion of the argument.

Hope this helps!
 lsat2016
  • Posts: 59
  • Joined: May 29, 2016
|
#25883
Nikki Siclunov wrote:Hi lsat2016,

Answer choice (E), by contrast, has no relation to this causal argument. Maybe workplace safety conditions have improved across all industries - so what? This could be the result of the same legislation that the author believes improved worker safety in the high-risk industries. If anything, since the legislation does not appear to target high-risk industries in particular, answer choice (E) can easily be interpreted as strengthening the conclusion of the argument.

Hope this helps!

Hello,

I see how E would strengthen if the workplace safety conditions were improved by the legislation mentioned in the passage, but isn’t it also equally likely that some other legislation improved the working conditions across all industries? I thought E was talking about some other broad legislation and thought this answer choice would weaken because it weakens the strength of the legislation mentioned in the passage.

Thank you so much!!
 Nikki Siclunov
PowerScore Staff
  • PowerScore Staff
  • Posts: 1362
  • Joined: Aug 02, 2011
|
#25929
Of course! We just don't know what was responsible for the observed effect. All answer choice (E) shows is that the observed effect is even more extensive than what the stimulus seems to suggest. This could be due to a host of reasons, none of which are presented in answer choice (E). It's entirely possible that, since the legislation's intended coverage wasn't limited to high-risk industries, the observed improvement across all industries resulted from that legislation. Of course, anything is possible - which is precisely why answer choice (E) is incorrect. Even if you don't see it as a Strengthen answer, it surely doesn't weaken the argument.

Thanks,

Get the most out of your LSAT Prep Plus subscription.

Analyze and track your performance with our Testing and Analytics Package.